Finding Density Function of Transformation












0















Let $Y$ be uniformly distributed over the interval $(−1, 3)$. Find the probability density function
of $U = Y^2$.




Attempted solution:



We know that



$$
Y=begin{cases}
1/4 && -1 leq y leq 3\
0 && text{elsewhere}
end{cases}
$$



We find the CDF of $U$ using $Y$
$$
P(U leq u) = P(Y^2 leq u) = P(-sqrt u leq Y leq sqrt u)
$$

$$
= int_{-sqrt u}^{sqrt u}frac{1}{4}dy = frac{sqrt u}{2}
$$



Therefore



$$
f(u) = frac{d}{du}frac{sqrt u}{2} = frac{1}{4sqrt u} qquad 0 leq u leq 9
$$



However, the answer is:



$$
f(u) = begin{cases} frac{1}{4sqrt u} && 0 leq u < 1\
frac{1}{8sqrt u} && 1 leq u leq 9
end{cases}
$$



Logically, since Y is between -1 and 3, this makes sense to me since there are two values of Y for which U will be between 0 and 1 for each value, and only one value of Y for which U is between 1 and 9.



Despite this, why didn't the computation pick up on this? Where did I go wrong?










share|cite|improve this question
























  • The statement of the problem, that you apparently copied, says "U= Y" but your calculation uses "$U= Y^2$. Which is it?
    – user247327
    Jan 3 at 22:42












  • My mistake. It should be $U = Y^2$. Sorry.
    – Bryden C
    Jan 3 at 22:44










  • The square function is not injective on $(-1,3)$ so taking the inversion as you do requires some care.
    – LoveTooNap29
    Jan 3 at 22:51
















0















Let $Y$ be uniformly distributed over the interval $(−1, 3)$. Find the probability density function
of $U = Y^2$.




Attempted solution:



We know that



$$
Y=begin{cases}
1/4 && -1 leq y leq 3\
0 && text{elsewhere}
end{cases}
$$



We find the CDF of $U$ using $Y$
$$
P(U leq u) = P(Y^2 leq u) = P(-sqrt u leq Y leq sqrt u)
$$

$$
= int_{-sqrt u}^{sqrt u}frac{1}{4}dy = frac{sqrt u}{2}
$$



Therefore



$$
f(u) = frac{d}{du}frac{sqrt u}{2} = frac{1}{4sqrt u} qquad 0 leq u leq 9
$$



However, the answer is:



$$
f(u) = begin{cases} frac{1}{4sqrt u} && 0 leq u < 1\
frac{1}{8sqrt u} && 1 leq u leq 9
end{cases}
$$



Logically, since Y is between -1 and 3, this makes sense to me since there are two values of Y for which U will be between 0 and 1 for each value, and only one value of Y for which U is between 1 and 9.



Despite this, why didn't the computation pick up on this? Where did I go wrong?










share|cite|improve this question
























  • The statement of the problem, that you apparently copied, says "U= Y" but your calculation uses "$U= Y^2$. Which is it?
    – user247327
    Jan 3 at 22:42












  • My mistake. It should be $U = Y^2$. Sorry.
    – Bryden C
    Jan 3 at 22:44










  • The square function is not injective on $(-1,3)$ so taking the inversion as you do requires some care.
    – LoveTooNap29
    Jan 3 at 22:51














0












0








0








Let $Y$ be uniformly distributed over the interval $(−1, 3)$. Find the probability density function
of $U = Y^2$.




Attempted solution:



We know that



$$
Y=begin{cases}
1/4 && -1 leq y leq 3\
0 && text{elsewhere}
end{cases}
$$



We find the CDF of $U$ using $Y$
$$
P(U leq u) = P(Y^2 leq u) = P(-sqrt u leq Y leq sqrt u)
$$

$$
= int_{-sqrt u}^{sqrt u}frac{1}{4}dy = frac{sqrt u}{2}
$$



Therefore



$$
f(u) = frac{d}{du}frac{sqrt u}{2} = frac{1}{4sqrt u} qquad 0 leq u leq 9
$$



However, the answer is:



$$
f(u) = begin{cases} frac{1}{4sqrt u} && 0 leq u < 1\
frac{1}{8sqrt u} && 1 leq u leq 9
end{cases}
$$



Logically, since Y is between -1 and 3, this makes sense to me since there are two values of Y for which U will be between 0 and 1 for each value, and only one value of Y for which U is between 1 and 9.



Despite this, why didn't the computation pick up on this? Where did I go wrong?










share|cite|improve this question
















Let $Y$ be uniformly distributed over the interval $(−1, 3)$. Find the probability density function
of $U = Y^2$.




Attempted solution:



We know that



$$
Y=begin{cases}
1/4 && -1 leq y leq 3\
0 && text{elsewhere}
end{cases}
$$



We find the CDF of $U$ using $Y$
$$
P(U leq u) = P(Y^2 leq u) = P(-sqrt u leq Y leq sqrt u)
$$

$$
= int_{-sqrt u}^{sqrt u}frac{1}{4}dy = frac{sqrt u}{2}
$$



Therefore



$$
f(u) = frac{d}{du}frac{sqrt u}{2} = frac{1}{4sqrt u} qquad 0 leq u leq 9
$$



However, the answer is:



$$
f(u) = begin{cases} frac{1}{4sqrt u} && 0 leq u < 1\
frac{1}{8sqrt u} && 1 leq u leq 9
end{cases}
$$



Logically, since Y is between -1 and 3, this makes sense to me since there are two values of Y for which U will be between 0 and 1 for each value, and only one value of Y for which U is between 1 and 9.



Despite this, why didn't the computation pick up on this? Where did I go wrong?







probability-distributions






share|cite|improve this question















share|cite|improve this question













share|cite|improve this question




share|cite|improve this question








edited Jan 3 at 22:43

























asked Jan 3 at 22:35









Bryden C

30418




30418












  • The statement of the problem, that you apparently copied, says "U= Y" but your calculation uses "$U= Y^2$. Which is it?
    – user247327
    Jan 3 at 22:42












  • My mistake. It should be $U = Y^2$. Sorry.
    – Bryden C
    Jan 3 at 22:44










  • The square function is not injective on $(-1,3)$ so taking the inversion as you do requires some care.
    – LoveTooNap29
    Jan 3 at 22:51


















  • The statement of the problem, that you apparently copied, says "U= Y" but your calculation uses "$U= Y^2$. Which is it?
    – user247327
    Jan 3 at 22:42












  • My mistake. It should be $U = Y^2$. Sorry.
    – Bryden C
    Jan 3 at 22:44










  • The square function is not injective on $(-1,3)$ so taking the inversion as you do requires some care.
    – LoveTooNap29
    Jan 3 at 22:51
















The statement of the problem, that you apparently copied, says "U= Y" but your calculation uses "$U= Y^2$. Which is it?
– user247327
Jan 3 at 22:42






The statement of the problem, that you apparently copied, says "U= Y" but your calculation uses "$U= Y^2$. Which is it?
– user247327
Jan 3 at 22:42














My mistake. It should be $U = Y^2$. Sorry.
– Bryden C
Jan 3 at 22:44




My mistake. It should be $U = Y^2$. Sorry.
– Bryden C
Jan 3 at 22:44












The square function is not injective on $(-1,3)$ so taking the inversion as you do requires some care.
– LoveTooNap29
Jan 3 at 22:51




The square function is not injective on $(-1,3)$ so taking the inversion as you do requires some care.
– LoveTooNap29
Jan 3 at 22:51










1 Answer
1






active

oldest

votes


















1














If $1<u<9$, then



$$P(-sqrt{u}leq Y < sqrt{u}) = P(-1leq Y leq sqrt{u}) = int_{-1}^{sqrt{u}} 1/4 dy $$



This should solve your problem. Remember that Y has 0 density outside of the interval [-1,3].






share|cite|improve this answer








New contributor




Leander Tilsted Kristensen is a new contributor to this site. Take care in asking for clarification, commenting, and answering.
Check out our Code of Conduct.


















  • Wouldn't $Y=0$ satisfy your second probability range? If so, then $U = Y^2 = 0$ but $U not in (1,9)$.
    – Bryden C
    Jan 3 at 22:55












  • @BrydenC What you are saying does not make the answer wrong. The answer given by Leander Tilsted Kristensen is correct.
    – Kavi Rama Murthy
    2 days ago











Your Answer





StackExchange.ifUsing("editor", function () {
return StackExchange.using("mathjaxEditing", function () {
StackExchange.MarkdownEditor.creationCallbacks.add(function (editor, postfix) {
StackExchange.mathjaxEditing.prepareWmdForMathJax(editor, postfix, [["$", "$"], ["\\(","\\)"]]);
});
});
}, "mathjax-editing");

StackExchange.ready(function() {
var channelOptions = {
tags: "".split(" "),
id: "69"
};
initTagRenderer("".split(" "), "".split(" "), channelOptions);

StackExchange.using("externalEditor", function() {
// Have to fire editor after snippets, if snippets enabled
if (StackExchange.settings.snippets.snippetsEnabled) {
StackExchange.using("snippets", function() {
createEditor();
});
}
else {
createEditor();
}
});

function createEditor() {
StackExchange.prepareEditor({
heartbeatType: 'answer',
autoActivateHeartbeat: false,
convertImagesToLinks: true,
noModals: true,
showLowRepImageUploadWarning: true,
reputationToPostImages: 10,
bindNavPrevention: true,
postfix: "",
imageUploader: {
brandingHtml: "Powered by u003ca class="icon-imgur-white" href="https://imgur.com/"u003eu003c/au003e",
contentPolicyHtml: "User contributions licensed under u003ca href="https://creativecommons.org/licenses/by-sa/3.0/"u003ecc by-sa 3.0 with attribution requiredu003c/au003e u003ca href="https://stackoverflow.com/legal/content-policy"u003e(content policy)u003c/au003e",
allowUrls: true
},
noCode: true, onDemand: true,
discardSelector: ".discard-answer"
,immediatelyShowMarkdownHelp:true
});


}
});














draft saved

draft discarded


















StackExchange.ready(
function () {
StackExchange.openid.initPostLogin('.new-post-login', 'https%3a%2f%2fmath.stackexchange.com%2fquestions%2f3061099%2ffinding-density-function-of-transformation%23new-answer', 'question_page');
}
);

Post as a guest















Required, but never shown

























1 Answer
1






active

oldest

votes








1 Answer
1






active

oldest

votes









active

oldest

votes






active

oldest

votes









1














If $1<u<9$, then



$$P(-sqrt{u}leq Y < sqrt{u}) = P(-1leq Y leq sqrt{u}) = int_{-1}^{sqrt{u}} 1/4 dy $$



This should solve your problem. Remember that Y has 0 density outside of the interval [-1,3].






share|cite|improve this answer








New contributor




Leander Tilsted Kristensen is a new contributor to this site. Take care in asking for clarification, commenting, and answering.
Check out our Code of Conduct.


















  • Wouldn't $Y=0$ satisfy your second probability range? If so, then $U = Y^2 = 0$ but $U not in (1,9)$.
    – Bryden C
    Jan 3 at 22:55












  • @BrydenC What you are saying does not make the answer wrong. The answer given by Leander Tilsted Kristensen is correct.
    – Kavi Rama Murthy
    2 days ago
















1














If $1<u<9$, then



$$P(-sqrt{u}leq Y < sqrt{u}) = P(-1leq Y leq sqrt{u}) = int_{-1}^{sqrt{u}} 1/4 dy $$



This should solve your problem. Remember that Y has 0 density outside of the interval [-1,3].






share|cite|improve this answer








New contributor




Leander Tilsted Kristensen is a new contributor to this site. Take care in asking for clarification, commenting, and answering.
Check out our Code of Conduct.


















  • Wouldn't $Y=0$ satisfy your second probability range? If so, then $U = Y^2 = 0$ but $U not in (1,9)$.
    – Bryden C
    Jan 3 at 22:55












  • @BrydenC What you are saying does not make the answer wrong. The answer given by Leander Tilsted Kristensen is correct.
    – Kavi Rama Murthy
    2 days ago














1












1








1






If $1<u<9$, then



$$P(-sqrt{u}leq Y < sqrt{u}) = P(-1leq Y leq sqrt{u}) = int_{-1}^{sqrt{u}} 1/4 dy $$



This should solve your problem. Remember that Y has 0 density outside of the interval [-1,3].






share|cite|improve this answer








New contributor




Leander Tilsted Kristensen is a new contributor to this site. Take care in asking for clarification, commenting, and answering.
Check out our Code of Conduct.









If $1<u<9$, then



$$P(-sqrt{u}leq Y < sqrt{u}) = P(-1leq Y leq sqrt{u}) = int_{-1}^{sqrt{u}} 1/4 dy $$



This should solve your problem. Remember that Y has 0 density outside of the interval [-1,3].







share|cite|improve this answer








New contributor




Leander Tilsted Kristensen is a new contributor to this site. Take care in asking for clarification, commenting, and answering.
Check out our Code of Conduct.









share|cite|improve this answer



share|cite|improve this answer






New contributor




Leander Tilsted Kristensen is a new contributor to this site. Take care in asking for clarification, commenting, and answering.
Check out our Code of Conduct.









answered Jan 3 at 22:52









Leander Tilsted Kristensen

112




112




New contributor




Leander Tilsted Kristensen is a new contributor to this site. Take care in asking for clarification, commenting, and answering.
Check out our Code of Conduct.





New contributor





Leander Tilsted Kristensen is a new contributor to this site. Take care in asking for clarification, commenting, and answering.
Check out our Code of Conduct.






Leander Tilsted Kristensen is a new contributor to this site. Take care in asking for clarification, commenting, and answering.
Check out our Code of Conduct.












  • Wouldn't $Y=0$ satisfy your second probability range? If so, then $U = Y^2 = 0$ but $U not in (1,9)$.
    – Bryden C
    Jan 3 at 22:55












  • @BrydenC What you are saying does not make the answer wrong. The answer given by Leander Tilsted Kristensen is correct.
    – Kavi Rama Murthy
    2 days ago


















  • Wouldn't $Y=0$ satisfy your second probability range? If so, then $U = Y^2 = 0$ but $U not in (1,9)$.
    – Bryden C
    Jan 3 at 22:55












  • @BrydenC What you are saying does not make the answer wrong. The answer given by Leander Tilsted Kristensen is correct.
    – Kavi Rama Murthy
    2 days ago
















Wouldn't $Y=0$ satisfy your second probability range? If so, then $U = Y^2 = 0$ but $U not in (1,9)$.
– Bryden C
Jan 3 at 22:55






Wouldn't $Y=0$ satisfy your second probability range? If so, then $U = Y^2 = 0$ but $U not in (1,9)$.
– Bryden C
Jan 3 at 22:55














@BrydenC What you are saying does not make the answer wrong. The answer given by Leander Tilsted Kristensen is correct.
– Kavi Rama Murthy
2 days ago




@BrydenC What you are saying does not make the answer wrong. The answer given by Leander Tilsted Kristensen is correct.
– Kavi Rama Murthy
2 days ago


















draft saved

draft discarded




















































Thanks for contributing an answer to Mathematics Stack Exchange!


  • Please be sure to answer the question. Provide details and share your research!

But avoid



  • Asking for help, clarification, or responding to other answers.

  • Making statements based on opinion; back them up with references or personal experience.


Use MathJax to format equations. MathJax reference.


To learn more, see our tips on writing great answers.





Some of your past answers have not been well-received, and you're in danger of being blocked from answering.


Please pay close attention to the following guidance:


  • Please be sure to answer the question. Provide details and share your research!

But avoid



  • Asking for help, clarification, or responding to other answers.

  • Making statements based on opinion; back them up with references or personal experience.


To learn more, see our tips on writing great answers.




draft saved


draft discarded














StackExchange.ready(
function () {
StackExchange.openid.initPostLogin('.new-post-login', 'https%3a%2f%2fmath.stackexchange.com%2fquestions%2f3061099%2ffinding-density-function-of-transformation%23new-answer', 'question_page');
}
);

Post as a guest















Required, but never shown





















































Required, but never shown














Required, but never shown












Required, but never shown







Required, but never shown

































Required, but never shown














Required, but never shown












Required, but never shown







Required, but never shown







Popular posts from this blog

An IMO inspired problem

Management

Has there ever been an instance of an active nuclear power plant within or near a war zone?